Constitutional Framework

Subscribe to our YouTube Channel – CivilsTap by EduTap for free, quality and regular content.

Historical Background of Indian Constitution ...... 2 Provisions adopted from constitutions of Developments during Company Rule (1773-1857) different countries ...... 12 ...... 2 Balancing Rights and duties of the citizens ...... 13 Regulating Act of 1773 ...... 2 Directive Principles of State Policy ...... 14 Pitt’s Act of 1784 ...... 2 Adopting Parliamentary form of Government ... 14 Charter act of 1793 ...... 2 Integrated and Independent Judiciary ...... 15 Charter act of 1813 ...... 2 Universal Adult Franchise ...... 16 Charter Act of 1833 ...... 3 System of Single Citizenship ...... 16 Charter Act of 1853 ...... 3 System of local government- Panchayats and Developments during Crown rule (1858-1947) ... 3 municipalities ...... 16 Government of India act 1858 ...... 3 Independent agencies as bulwarks of constitution ...... 16 Indian council act of 1861 ...... 4 Emergency provisions ...... 17 Indian Council Act of 1892 ...... 4 Schedules of Indian Constitution ...... 17 Indian Council Act of 1909 (Morley-minto reforms) 4 Preamble of Indian constitution ...... 18 Government of india Act of 1919 (Montagu- Chelmsford Reforms) ...... 5 Components of Preamble ...... 19 Government of India Act of 1935 ...... 5 Source of its authority ...... 19 Indian Independence act of 1947 ...... 6 Nature of Indian State ...... 19 Making of the Indian Constitution ...... 7 Date of the adoption of Indian Constitution ...... 19 Demand for Independent Constituent Assembly 7 Objectives of the Indian constitution ...... 19 Cabinet Mission Plan for the Constituent Important judgements related to preamble ...... 20 assembly ...... 7 Amendment of Indian Constitution ...... 21 Changes made by the Indian Independence Act of Types of Amendments...... 21 1947 ...... 8 Procedure for amendment of Indian Constitution Working of the constituent Assembly ...... 8 ...... 22 Important Facts ...... 8 Issues with the amendment procedure ...... 23 Objective Resolution ...... 8 Content of Amendments made so far ...... 23 Committees of the Constituent Assembly ...... 9 Doctrine of Basic Structure of the Indian Deliberations and Finalisation of Indian Constitution Constitution ...... 24 ...... 9 Emergence of this Doctrine ...... 24 Other functions performed by constituent assembly What is the doctrine of Basic Structure? ...... 25 ...... 10 MCQs for Practice ...... 26 Criticism of Constituent Assembly ...... 10 Answers & Explanations ...... 30 Features of indian constitution ...... 11 Lengthiest and bulkiest written constitution of the world ...... 11 Constitution as living document ...... 11 Mixture of Federalism and Unitarianism ...... 11 Visit: www.civilstap.com For Any Query Mail us: [email protected] or call us at - (+91)-8146207241 1 Subscribe to our YouTube Channel – CivilsTap by EduTap for free, quality and regular content.

. It restricted the Court of Directors to four-year HISTORICAL BACKGROUND terms. OF INDIAN CONSTITUTION The Act did not prove to be a long-term solution to concerns over the Company's affairs despite the . Basic framework of the Indian Constitution amending act of 1781, known as the Act of traces its legacy to British rule in India. Settlement. It provided for: . Various developments during British rule starting . Exemption of the Governor-General and his from Regulating Act, 1773 up to the Government Council from the jurisdiction of Supreme Court. of India Act, 1935 have greatly influenced our . It empowered the Governor-General Council to Constitution and Polity. Let us look at these frame regulations for the Provincial courts and developments in chronological order. councils.

DEVELOPMENTS DURING COMPANY Further, Pitt's India Act was subsequently enacted in RULE (1773-1857) 1784 as a more radical reform.

During this period, East India Company (EIC) handled PITT’S INDIA ACT OF 1784 the Indian affairs under the overall supervision of British Parliament. SEPARATING COMMERCIAL AND POLITICAL FUNCTIONS . It started the system of double government. REGULATING ACT OF 1773 Court of directors of EIC were allowed to manage the commercial/financial affairs, however a six It was for the first time that British Government member board of controllers was set up for tried to overhaul the affairs of East India Company’s political activities. Indian administration. It strengthened the Parliamentary control over Company. Important INCREASING CONTROL OVER EIC provisions of this Act were: . Board of Control was also empowered to supervise the civil, military and revenue affairs CHANGES IN ADMINISTRATION of British possessions in India. . Governor of Bengal was designated as the “Governor General of Bengal”. Warren Hastings CHANGES IN EXECUTIVE COUNCIL was the first such Governor General. . The Executive council of the Company in India . It created an Executive Council of four persons was reduced to three members. to assist Governor General. There were series of Charter Acts passed from 1793 BEGINNING OF CENTRALISATION to 1853 in an interval of 20 years each. . It subsumed the presidencies of Bombay and Madras under Bengal’s control. Thus it laid the CHARTER ACT OF 1793 foundations of Central Administration in India. ADMINISTRATIVE AND JUDICIAL REFORMS NEW JUDICIAL SYSTEM . The Governor-General was given more powers. . Supreme Court was established at Fort William He could override his council’s decision under at Calcutta. It comprised of one chief justice and certain circumstances. three other judges. . This Act separated the revenue administration and the judiciary functions of the Company. REFORMS IN EIC . It prohibited the employees of East India CHARTER ACT OF 1813 Company from engaging in any private/personal trade and accepting gifts or bribes from locals. Charter act of 1813 ended the monopoly of the EIC in India, however the company’s monopoly in trade Visit: www.civilstap.com For Any Query Mail us: [email protected] or call us at - (+91)-8146207241 2 Subscribe to our YouTube Channel – CivilsTap by EduTap for free, quality and regular content. with China and trade in tea with India was kept BEGINNING OF LOCAL ASSOCIATION intact. . Local representation was introduced in the new Legislative council created under this Act. Four POWERS TO LOCAL GOVERNMENT members were appointed by the local . The Act also empowered the Local Governments (provincial) Governments of Madras, Bombay, in India to impose taxes on persons subject to the Bengal and Agra. jurisdiction of Supreme Court. This was for the first time, that this Charter Act, unlike other charter acts, did not fix any limit for the CHARTER ACT OF 1833 continuance of the administration of the company in . From now Governor General of Bengal was India. designated as “Governor General of India”. Lord In the wake of Revolt of 1857 (also known as the First William Bentick was the first Governor-General war of Independence) the company’s rule came to of India. an end. From now onwards the British crown . For the first time, the Governor-General's assumed the direct responsibility to govern India. Government was known as the 'Government of India' and his council as the 'India Council'. DEVELOPMENTS DURING CROWN RULE (1858-1947) CENTRALISATION AT ITS PEAK . The Governor-General and his executive council GOVERNMENT OF INDIA ACT 1858 were given exclusive legislative powers for the whole of British India. Thus it deprived the It was also known as the Act for Good Government Governor of Bombay and Madras of their of India. It made little changes in the system of legislative powers. administration prevalent in India, however the REFORMS IN EIC manner of supervision and control of Indian . It ended the activities of EIC as commercial body. administration in England was changed and It became an administrative body only. strengthened. . It attempted to introduce a system of open CHANGES IN ADMINISTRATION competitions in Civil Services examination. . The Company's territories in India were to be However this provision was negated after vested in the crown, the Company ceasing to opposition from the Court of Directors. exercise its power and control over these territories. CHARTER ACT OF 1853 . It ended the system of double Government by It was the final Act of Charter Act series. It marked abolishing the Board of Control and Court of the recognition of legislation as separate function of Directors. the Government. . The Crown was empowered to appoint a Governor-General and the Governors of the CREATION OF INDIAN LEGISLATIVE COUNCIL Presidencies. . It separated the executive and legislative functions of the Governor General’s Executive SECRETARY OF STATE AND HIS COUNCIL council. A separate Governor-General’s . New office of Secretary of State for India was legislative council was created through the created. The Secretary of State was a member of addition of six new members in the Executive the British cabinet and was responsible Council. ultimately to the British Parliament. . Also a new advisory body named, Council of CIVIL SERVICES OPENED TO INDIANS India was created to assist its chairman the . It introduced the system of open competition Secretary of State for India. for the selection and recruitment of Civil Servants in India. CHANGING DESIGNATION Visit: www.civilstap.com For Any Query Mail us: [email protected] or call us at - (+91)-8146207241 3 Subscribe to our YouTube Channel – CivilsTap by EduTap for free, quality and regular content.

. Governor General of India was now designated (b) Provincial legislative councils by the as Viceroy of India. Lord Canning was the first Provincial Governors on the recommendation of viceroy of India. the district boards, municipalities, universities, To smoothen the political setup in India and to seek trade associations, zamindars and chambers. the cooperation of Indians in administration three Acts were enacted by British Parliament in 1861, . It also increased the functions of legislative 1892 and 1909. council. It allowed councils to discuss (not vote on) each year's annual financial statement INDIAN COUNCIL ACT OF 1861 (Budget).

BEGINNING OF DECENTRALISATION . It restored the legislative powers of Bombay and QUICK FACT Madras Presidencies taken away by the Charter The system of Budget was introduced in British India in 1860 Act of 1833. Thus, it begun the process of decentralisation in Indian administration. Thus, whilst failing to answer calls for direct . It provided for the creation of legislative councils elections, the principle of representation was for Bengal, North-Western Frontier Province introduced in 1892 Act. (NWFP) and Punjab. INDIAN COUNCIL ACT OF 1909 INDIANS IN LEGISLATIVE COUNCIL (MORLEY-MINTO REFORMS) . It started the process of associating Indians in the law making process. It marked the beginning of This Act was enacted after the Swadeshi movement representative Institutions in India. (1905), formation of Muslim league (1906) and Surat NEW TOOLS OF IMPROVED EXECUTION split (1907) in Congress. There were huge . The ‘Portfolio’ System was introduced by Viceroy expectations among moderate Congress members Lord Canning. The council member’s holding any from this Act. particular departments were authorized to issue CHANGES IN LEGISLATIVE COUNCIL final orders on behalf of the council on matters . It established Indian dominance in the provincial, related to their department. but not central, legislative bodies. . The Viceroy was allowed to issue ordinances . It considerably increased the size of the lasting six months if the Legislative Council was legislative councils, both Central and provincial. not in session. The number of members in the Central Legislative Council was raised from 16 to 60. In INDIAN COUNCIL ACT OF 1892 provinces the increase was not uniform. CHANGES IN LEGISLATIVE COUNCIL . Members were also enabled to debate . The Act increased the number of additional budgetary and complementary matters and members to be represented in the Central and table resolutions. Provincial councils but maintained official RISE OF COMMUNALISM majority. For example, the number of Additional . It introduced separate electorates where seats Members elected to the Central Legislative were reserved for Muslims and in which only Council was increased to a range of ten to Muslims would be polled. It marked the sixteen. beginning of communal representation in India. . Some non-official members were to be

nominated in Satyendra Prasad Sinha became the first Indian to (a) Central Legislative Council by the viceroy on join the Viceroy’s Executive Council. He was the recommendation of the provincial legislative appointed as the law member. councils and the Bengal Chamber of Commerce,

and Visit: www.civilstap.com For Any Query Mail us: [email protected] or call us at - (+91)-8146207241 4 Subscribe to our YouTube Channel – CivilsTap by EduTap for free, quality and regular content.

GOVERNMENT OF INDIA ACT OF 1919 . It made the Central Legislature bicameral. The (MONTAGU-CHELMSFORD REFORMS) lower house was the Legislative Assembly, and the upper house was the Council of States. This Act had a separate Preamble which declared . The Franchise (Right of voting) was granted to that the objective of the British Government was the limited number of people on the basis of tax, gradual introduction of responsible Government in property and education. India. . Provincial budgets was separated from the REFORMS IN ADMINISTRATION Central budgets for the first time. . It demarcated Central and Provincial subjects. EXTENDING COMMUNALISM The Central and Provincial legislatures were . Principal of communal representation through authorised to make laws on their respective list separate electorate was extended to Sikhs, of subjects. Despite this relaxations, Government Anglo-Indians, Indian Christians and Europeans. structure continued to be centralised and unitary. NEW INSTITUTIONS . Diarchy was introduced at the Provincial level. . It provided for the establishment of a Public It means a dual set of Governments- one is Service Commission in India for the first time. accountable and other is not. The provincial . Some of the functions of Secretary of State was subjects were divided into transferred and transferred to new office of High Commissioner reserved subjects. The transferred subjects were for India. to be administered by Governor with the aid of ministers accountable to legislative councils. The It made a provision that a Statutory Commission reserved subjects were to be administered by would be set up at the end of 10 years after the act Governor without aid of ministers. to inquire into the working system of the . Except Commander in Chief, three out of six government. The Simon commission of 1927 was an member of viceroy’s executive council needed outcome of this provision. to be Indian. GOVERNMENT OF INDIA ACT OF 1935

QUESTION 1 This Act was passed in the backdrop of Simon Q. In the context of Indian history, the principle Commission (1927), three round table conferences of ‘Dyarchy (diarchy)’ refers to (UPSC 2017) and Communal Award (1932). It was the lengthiest (a) Division of the Central legislature into two of all Acts and aimed to establish responsible houses. Government in India. (b) Introduction of double Government i.e., INDIAN FEDERATION Central and State governments. . It provided for the establishment of an All-India (c) Having two sets of rulers; one in and Federation consisting of provinces and princely another in Delhi. states as units. (d) Division of the subjects delegated to the provinces into two categories. CHANGES IN ADMINISTRATION . It distributed the powers between Centre and Answer: D provinces through three lists; federal, provincial and concurrent lists. Residuary powers were given to the Viceroy. LEGISLATIVE CHANGES . Diarchy was introduced in the Centre. However . No Bill of the legislature could be deemed to it did not come into operation. have been passed unless assented to by the . It abolished the Council of India, established by Viceroy. The latter could however enact a Bill the Government of India Act of 1858. The without the assent of the legislature.

Visit: www.civilstap.com For Any Query Mail us: [email protected] or call us at - (+91)-8146207241 5 Subscribe to our YouTube Channel – CivilsTap by EduTap for free, quality and regular content.

secretary of state for India was provided with a Separate electorates for depressed classes team of advisors in its place. (scheduled castes), women and labour (workers) were introduced.

QUESTION 2 Q. The distribution of powers between the QUESTION 3 Centre and the States in the Indian Constitution Q. Which one of the following is not a feature of is based on the scheme provided in the ( UPSC Government of India Act of 1935? (UPSC 2000) 2012) (a) Diarchy at the Centre as well as in the (a) Morley-Minto Reforms, 1909 Provinces (b) Montagu-Chelmsford Act, 1919 (b) A bicameral legislature (c) Government of India Act, 1935 (c) Provincial autonomy (d) Indian Independence Act, 1947 (d) An All-India Federation

Answer: C Answer: A

DECENTRALISATION AT ITS PEAK INDIAN INDEPENDENCE ACT OF 1947 . It abolished diarchy in the provinces. It provided for provincial autonomy. It was the final Act of the British Parliament for India. . It introduced responsible Governments in It was enacted to give effect to Mountbatten plan provinces, that is, the Governor was required to (June, 1947). act with the advice of ministers responsible to the provincial legislature. It was discontinued INDEPENDENCE WITH PARTITION after 1939. . It provided for the Independence of India and ended the British rule. LEGISLATIVE CHANGES . It provided for two dominion states: India and . Bicameralism was introduced in six out of Pakistan. eleven provincial legislature. Bengal, Bombay, . It provided for partition of Punjab & Bengal and Madras, Bihar, Assam and the United Provinces separate boundary commissions to demarcate were made bicameral. the boundaries between them. . This Act introduced for the first time the direct . It provided, for each dominion, a Governor- elections. About 10% of the total population got General, who was to be appointed by the British the voting rights. King on the advice of the dominion cabinet. . It proclaimed the lapse of British paramountcy NEW INSTITUTIONS . This Act vested the control of Railway in a new over the Indian princely states and treaty authority called Federal Railway Authority, relations with tribal areas from August 15, 1947. which was free from control of ministers. . It abolished the office of the Secretary of state . It provided for the establishment of Federal for India. Court to interpret the Act and adjudicate CONSTITUTIONAL ADVANCEMENT disputes relating to the federal matters. . The Constituent assemblies of both the . It provided for the establishment of Provincial dominions were free to make Constitutions of Public Service Commission and Joint Public their respective countries. Service Commission for two or more provinces. . For the time being till the Constitution was made, . It provided for the establishment of a Reserve both of them would be governed in accordance Bank of India. with the Government of India Act 1935.

Visit: www.civilstap.com For Any Query Mail us: [email protected] or call us at - (+91)-8146207241 6 Subscribe to our YouTube Channel – CivilsTap by EduTap for free, quality and regular content.

. Those Civil servants who had been appointed . From the British side, August Offer (August, before the August 15, 1947, will continue in 1940) made the first in principle approval of the service with same privileges. constituent assembly, where mainly Indians would decide their constitution. On 15th August, 1947 British rule came to an end. . Cripps Mission (March, 1942) also provided for India became independent and Constituent establishment of Constituent assembly. This idea Assembly of India formed in 1946 became the was rejected by both Congress and Muslim Parliament of the Indian Dominion. league due to various differences. . Finally Cabinet Mission (May, 1946) provided a plan for the establishment of Constituent MAKING OF THE INDIAN assembly which more or less satisfied Congress CONSTITUTION as well as Muslim league.

Indian Constitution was made by the Constituent CABINET MISSION PLAN FOR THE Assembly elected indirectly by members of CONSTITUENT ASSEMBLY provincial legislative assemblies that had been established under the Government of India Act, COMPOSITION OF CONSTITUENT ASSEMBLY 1935. Initially it had members from undivided India . Strength- There would be 389 seats in but after partition only members from Indian Constituent assembly having members from dominion were present. British provinces as well as Princely States. . It included fifteen women. Prominent ones were DEMAND FOR INDEPENDENT Hansa Mehta, Renuka Ray and Begum Aizaz CONSTITUENT ASSEMBLY Rasul.

Although the making of began SEAT ALLOCATION in December 1946, the demand for Constitution of . Out of 393, 296 seats were allotted to British India by independent Indian constituent assembly India and 93 seats to the Princely States. started way back. . Further under 296 seats allotted to the British India, 292 members were to be drawn from the NATIONALIST’S DEMAND eleven governors’ provinces and four from the . In 1922, , in an article titled four chief commissioners’ provinces, one from “Independence” published in Young India, each. indicated that the Constitution of India would be . The seats in each Province were distributed drafted by the Indians. among the three main communities, Muslims, . In response to Lord Birkenhead’s challenge, the Sikhs and general, in proportion to their Nehru report, submitted on August 1928, was an respective populations outline of draft Constitution for India. . Each Province and each Princely State or group . In 1934 the idea of a Constituent Assembly for of States were allotted seats proportional to India was put forward for the first time by M. N. their respective population roughly in the ratio Roy, a pioneer of communist movement in India. of 1:10,00,000. . In 1935, the Indian National Congress (INC), for the first time, officially demanded a Constituent ELECTION METHOD assembly to frame the Constitution of India. . Members of each community in the Provincial . The same demand was reiterated in the Haripura Legislative Assembly elected their own session of 1938. representatives by the method of proportional representation with single transferable vote. BRITISH RESPONSE

Visit: www.civilstap.com For Any Query Mail us: [email protected] or call us at - (+91)-8146207241 7 Subscribe to our YouTube Channel – CivilsTap by EduTap for free, quality and regular content.

. The representatives of princely states were to be . Constitutional advisor- Sir B.N.Rau nominated by the heads of the princely states. . Symbol (seal)- Elephant Thus, final Constituent Assembly was to be a partly . Secretary - H.V.R. Iyengar elected and partly nominated body. The . Calligrapher- Prem Behari Narain Raizada representatives of the princely states, generally . Chief draftsman of the constitution- S.N. stayed away from the Constituent Assembly until Mukerjee Independence. . In all, the Constituent Assembly had 11 sessions over two years, 11 months and 18 days. CHANGES MADE BY THE INDIAN . The total expenditure incurred on making the INDEPENDENCE ACT OF 1947 Constitution amounted to ` 64 lakh. . Mahatma Gandhi was not the part of As per the Act, India became independent but with constituent assembly. partition. The Constitution was thus framed against

the backdrop of the horrendous violence that the The original version was beautified and decorated by partition unleashed on the subcontinent. This Act led artists from including Nand Lal Bose to the following changes in the Constituent Assembly and Beohar Rammanohar Sinha. established in 1946. . Constituent Assembly from now became a fully OBJECTIVE RESOLUTION sovereign body, which could frame any Constitution it pleased. The philosophy underlying the Indian Constitution . As a consequence of the Partition, those was embodied in the Objective Resolution. Main members who were elected from territories points related to it are: which fell under Pakistan ceased to be members of the Constituent Assembly. MOVED BY It was moved in the Constituent Assembly on . The number of members in the Assembly was December 13, 1946 by Pt. and reduced to 299. Out of these, 229 were from adopted after considerable debate and deliberation British India and 70 from princely states. in the Assembly on January 22, 1947. . The Constituent Assembly also became a legislative body (first Parliament of free India). FEATURES . The representatives of most of the princely . India is an independent, sovereign, republic; states took their seats in the Assembly. . India shall be a Union of erstwhile British Indian territories, Indian States, and other parts outside British India and Indian States as are willing to be WORKING OF THE CONSTITUENT a part of the Union; ASSEMBLY . All powers and authority of sovereign and independent India and its constitution shall flow Constituent assembly held its first meeting on from the people; December 9, 1946 with 211 members attending. . All people of India shall be guaranteed and Muslim League did not attend it. secured social, economic and political justice; equality of status and opportunities and equality IMPORTANT FACTS before law; and fundamental freedoms - of . Dr Sachchidanand Sinha, the oldest member, was speech, expression, belief, faith, worship, elected as the temporary President of the Assembly vocation, association and action subject to law (French practice). and public morality; . . Elected President- Dr. The minorities, backward and tribal areas, . Vice President-Two were there, H.C.Mukherjee depressed and other backward classes shall be and V.T.Krishnamachari provided adequate safeguards;

Visit: www.civilstap.com For Any Query Mail us: [email protected] or call us at - (+91)-8146207241 8 Subscribe to our YouTube Channel – CivilsTap by EduTap for free, quality and regular content.

. The land would make full and willing contribution It was one of the most important committee of to the promotion of world peace and welfare of constituent assembly headed by Dr. B.R.Ambedkar mankind; and assisted by Sir B.N.Rau. . The territorial integrity of the Republic and its . It was setup on 29 August, 1947 sovereign rights on land, sea and air shall be . It consisted of 6 other members namely N maintained according to justice and law of Gopalaswamy Ayyangar, Alladi Krishnaswamy civilized nations; Ayyar, Dr K M Munshi, Syed Mohammad Modified version of Objective resolution forms the Saadullah, N Madhava Rau, T.T. Krishnamachari Preamble of the present Constitution. . Draft Constitution was prepared by the committee and submitted to the Assembly on 4 COMMITTEES OF THE CONSTITUENT November 1948. It was debated and over 2000 ASSEMBLY amendments were moved over a period of two years. Constituent Assembly appointed both major as well as minor committees to deal with various tasks of Constitution making. Each Committee usually QUESTION 4 drafted particular provisions of the Constitution Q. Who among the following was the chairman of which were then subjected to debate by the entire the Union Constitution Committee of the Assembly. Constituent Assembly? (UPSC 2005) (a) Dr B.R. Ambedkar MAJOR COMMITTEES (b) J.B Kripalani

(c) Pt Jawaharlal Nehru Committee Headed by 1. Union Powers Committee Jawaharlal Nehru (d) Alladi Krishnaswami Ayyar 2. Union Constitution Committee Jawaharlal Nehru 3. States Committee Jawaharlal Nehru Answer: C 4. Drafting Committee Dr. B.R.Ambedkar 5. Rules of Procedure Committee Dr. Rajendra Prasad 6. Steering Committee Dr. Rajendra Prasad DELIBERATIONS AND FINALISATION 7. Provincial Constitution Committee Sardar Patel OF INDIAN CONSTITUTION 8.Advisory Committee on Sardar Patel Fundamental Rights, Minorities, Each member deliberated upon the Constitution Tribal and Excluded Areas with the interests of the whole nation in mind.

Almost every issue that lies at the foundation of a MINOR COMMITTEES modern state was discussed with great They were headed by both assembly members and sophistication. non-assembly members. Few among the important . The members of the Assembly placed a great minor committees were emphasis on discussion and reasoned argument. . Some provisions were subject to the vote. Committee Headed by . Only one provision of the Constitution was 1. Finance and Staff Committee Dr. Rajendra Prasad passed without virtually any debate: the 2. Credentials Committee Alladi Krishnaswami introduction of universal suffrage. Ayyar 3. Order of Business Committee Dr. K.M. Munshi . The sessions were open to the press and the 4. Press Gallery Committee Usha Nath Sen public alike. 5. Linguistic Provinces Commission S.K. Dar ENACTMENT OF THE CONSTITUTION 6. House Committee B. P. Sitaramayya 7. Committee on Citizenship S. Varadachari . The Final reading of the draft started on November 14, 1949. The motion on Draft DRAFTING COMMITTEE Constitution was declared as passed on

Visit: www.civilstap.com For Any Query Mail us: [email protected] or call us at - (+91)-8146207241 9 Subscribe to our YouTube Channel – CivilsTap by EduTap for free, quality and regular content.

November 26, 1949, and received the signatures possibility to change the provisions and the limits of the members and the president. on such changes, the Constitution has ensured . 26 November, 1949 is mentioned in the that it will survive as a document respected by Preamble as the date on which the people of people. India in the Constituent Assembly adopted, enacted and gave to themselves this OTHER FUNCTIONS PERFORMED BY Constitution. CONSTITUENT ASSEMBLY . The Constitution as adopted on November 26, Constituent assembly also performed the following 1949, contained a Preamble, 395 Articles and 8 important functions; Schedules. . It adopted the National Flag on July 22, 1947 and

the National Anthem and National Song on Dr. B.R.Ambedkar is recognised as the ‘Father of the January 24, 1950. Constitution of India’ and “Modern Manu” for his . It elected Dr Rajendra Prasad as the first contribution in the enactment of Constitution. President of India on January 24, 1950. ENFORCEMENT OF THE CONSTITUTION . It ratified India’s membership of the Commonwealth in May 1949. ON 26TH NOVEMBER, 1949 . Drafting of ordinary legislation for the country. Some provisions relating to citizenship, elections, For the first time, the Constituent Assembly met provisional parliament, temporary and transitional as Dominion Legislature on November 17, 1947 provisions, and short title in the Indian Constitution and elected G V Mavlankar as its speaker. came into force on November 26, 1949 itself.

ON 26TH JANUARY, 1950 CRITICISM OF CONSTITUENT . The Assembly met once again on 24 January, ASSEMBLY 1950, when the members appended their signatures to the Constitution of India Every great step in the history is followed by the . Major part of the constitution containing criticism of the same. In the same manner, the remaining provisions came into force on 26th Constituent Assembly was also criticized. Some of January, 1950. the points included: . The Constitution of India came into force on 26 . Constituent Assembly was not a representative January, 1950. On that day, the Assembly ceased body as its members were not directly elected by to exist, transforming itself into the Provisional the people of India. Parliament of India until a new Parliament was . As it was created on the proposals of the British constituted in1952. Government, some people didn’t consider it as a Sovereign body. AUTHORITY OF THE CONSTITUTION . When compared it to the framers of the The Constitution drew enormous legitimacy from American Constitution, who constituted the the fact that American Constitution in 4 months, the . It was drawn up by people who enjoyed constituent assembly took too much time. immense public credibility, who had the capacity . Critics stated it as a “Lawyer-Politician to negotiate and command the respect of a wide Domination.” cross-section of society. . It was dominated by Congress. . In a way, the Constituent Assembly was giving . According to some critics, the Constituent concrete shape and form to the principles it had Assembly was a Hindu dominated body. inherited from the nationalist movement.

. Indian Constitution gives everyone a reason to go

along with its broad outlines.

. The Indian Constitution is described as ‘a living’

document. By striking a balance between the Visit: www.civilstap.com For Any Query Mail us: [email protected] or call us at - (+91)-8146207241 10 Subscribe to our YouTube Channel – CivilsTap by EduTap for free, quality and regular content.

FEATURES OF INDIAN QUESTION 5 Q. Consider the following statements CONSTITUTION 1. The Constitution of India has 40 parts. Constitution is the Fundamental Law of any country 2. There are 390 Articles in the Constitution of which sets out the framework and the principal India in all. functions of various organs of the Government. 3. Ninth, Tenth, Eleventh and Twelfth Schedules Indian constitution has following features which were added to the Constitution of India by make it unique in terms of content and spirit. Constitution (Amendment) Acts. Which of the statement(s) given above is/are LENGTHIEST AND BULKIEST correct? (UPSC 2005) WRITTEN CONSTITUTION OF THE (a) 1 and 2 (b) Only 2 WORLD (c) Only 3 (d) 1, 2 and 3

The Indian Constitution is a wholly written Answer: C document. . The Constitution as adopted on November 26, 1949, contained a Preamble, 395 Articles and 8 CONSTITUTION AS LIVING Schedules. . More than 100 amendments since its adoption DOCUMENT further increased its size. Our Constitution is not a static document, it is not . At present it consists of a Preamble, about 465 the final word about everything; it is not unalterable. Articles (divided into 25 Parts) and 12 Schedules. . Indian Constitution presents a beautiful blend of . flexibility and rigidity. . Flexibility means some of its provisions can be amended in a manner similar to ordinary laws. It 1. About 90 Articles 1. About 20 Articles is for adopting to and managing the evolving 2. Four Schedules 2. One Part (VII) situations and to rectify mistakes. (9,10,11 and 12) . Rigidity guarantees that those features which are 3. Four parts (IVA, central to the spirit of the Constitution should be IXA, IXB, XIVA) protected from change (Amendment procedures are discussed in later chapter).

FACTORS RESPONSIBLE FOR BULKINESS . Vast size and large diversity of India MIXTURE OF FEDERALISM AND . There is a single Constitution for Centre as well UNITARIANISM as States . Due to dominance of lawyers in Constituent assembly While describing India as a Union of States, the . Legacy of Government of India Act of 1935. Constitution provides for a federal structure with a . It contains administrative as well as governance unitary spirit. Scholars describe India as a ‘Quasi- provisions. Federation’ (K.C. Wheare) or as ‘a federation with a . It covers most of the parliamentary conventions unitary bias, or even as ‘a Unitarian federation.’ otherwise followed in unwritten way. LIKE A FEDERATION, THE CONSTITUTION OF INDIA PROVIDES FOR:

Visit: www.civilstap.com For Any Query Mail us: [email protected] or call us at - (+91)-8146207241 11 Subscribe to our YouTube Channel – CivilsTap by EduTap for free, quality and regular content.

. Dual polity consisting the Union at the Centre PROVISIONS ADOPTED FROM and the states at the periphery. . Structure, organisation, powers and functions of CONSTITUTIONS OF DIFFERENT both the Central and State Governments and COUNTRIES limits within which they must operate. . Division of Powers between centre and states. The Indian Constitution has borrowed provisions . Rigidity to the extent that provisions concerned from various countries and modified them according with the federal structure can be amended only to the suitability and requirements of the country. by the joint action of both. . Bicameralism consist of Rajya Sabha (represents the states of Indian Federation) and Lok Sabha. Structural Part . Independent judiciary with the power to decide from GoI Act, centre-state disputes. 1935 The framers adopted the federal features due to Political Part Philosphical vastness of the country and its socio-cultural Part from from British American diversity. It ensures the efficient governance of the Constitution country and also manages national unity with constitution regional autonomy.

UNITARY SPIRIT OF INDIAN CONSTITUTION IS Indian REFLECTED BY: Constitution . Inequitable distribution of power with strong inclination towards centre. . Single constitution for centre as well as states. . Flexibility provided to Parliament to amend bulk Various other provisions were drawn as follows; of the Constitution unilaterally. . There is unequal representation of states in the Sources Features borrowed Government of India 1. Federal Scheme Rajya Sabha, with Uttar Pradesh having 31 seats Act of 1935 2. Office of Governor and smaller states having only 1. 3. Judiciary . Emergency provisions. 4. Public Service Commissions . Presence of Integrated Judicial system. 5. Emergency provisions . Integrated audit and election machinery 6. Administrative details

. All India Services violate the principle of British Constitution 1. Parliamentary government, 2. Rule of Law federalism under the Constitution. 3. Legislative procedure 4. Single citizenship CONSTITUTIONAL STATUS 5. Cabinet system . As per the Article 1 of the Indian Constitution 6. Prerogative writs, “India is a Union of States’”. The term 7. Parliamentary privileges ‘federation’ has nowhere been used in the 8. Bicameralism Constitution. US Constitution 1. Fundamental rights

. According to Dr B R Ambedkar, the phrase ‘Union 2. Independence of judiciary 3. Judicial review of States’ has been preferred to ‘Federation of 4. Impeachment of the States’ to indicate two things: President (I) the states had not come into any agreement 5. Removal of Supreme Court to form the Indian federation like the American and high court judges federation; 6. Post of Vice president Irish Constitution 1. Directive Principles of State (II) No states have right to secede from the Policy federation. The federation is union because it is 2. Nomination of members to indestructible. Rajya Sabha Visit: www.civilstap.com For Any Query Mail us: [email protected] or call us at - (+91)-8146207241 12 Subscribe to our YouTube Channel – CivilsTap by EduTap for free, quality and regular content.

3. Method of election of Indian Constitution provides for the fundamental President rights to its citizens without any discrimination. Canadian Constitution 1. Federation with a strong These rights try to establish ideals of political Centre 2. Vesting of residuary powers democracy. in the Centre . Part III of Indian Constitution contains 3. Appointment of state exhaustive list of six fundamental rights governors by the Centre (originally seven) covering very vast scope. 4. Advisory jurisdiction of the These are; Supreme Court Germany Constitution Suspension of fundamental rights during emergency •Right to Equality Article Australia Constitution 1. Concurrent List 14-18 2. Freedom of trade, commerce and intercourse Article •Right to Freedom 3. Joint sitting of the two 19-22 Houses of Parliament Soviet Constitution 1. Fundamental duties Article •Right against exploitation 2. Ideal of justice in preamble 23-24 French Constitution 1. Republic

2. Ideals of liberty, equality and Article •Right to freedom of Religion fraternity 25-28 South African 1. Procedure for amendment of Constitution the Constitution Article •Cultural and Educational Rights 2. Election of members of Rajya 29-30 Sabha Japanese Constitution Procedure established by law Article •Right to Constitutional Remedies 32

QUESTION 6 . They uphold the principles of equality of all Q. Match List I with List II and select the correct individuals, the dignity of the individual, the answer (UPSC 2003) larger public interest and unity of the nation. LIST I- Items in Constitution LIST II- Source Country . They are defended and guaranteed by the A. Directive Principles of 1. Australia Supreme Court, hence, justiciable. However, State Policy B. Fundamental Rights 2. Canada they are not absolute but qualified. The State can C. Concurrent List 3. Ireland impose reasonable restrictions on them. D. India as Union of States 4. United Kingdom 5.USA FUNDAMENTAL DUTIES A set of ten fundamental duties were incorporated Select the correct answer using following Codes: in the Constitution through 42nd Constitutional A B C D Amendment Act of 1976. It ensures that citizens (a) 5 4 1 2 while enjoying fundamental rights also need to be (b) 5 4 2 1 conscious about their duties towards the state and (c) 3 4 2 1 fellow citizens. (d) 3 5 1 2 . These duties were included based on the

recommendations of Swaran Singh Committee. Answer: D . 86th Constitutional Amendment Act of 2002

added one more fundamental duty. BALANCING RIGHTS AND DUTIES OF . Article 51 A under Part IVA of the Indian Constitution contains 11 fundamental duties. THE CITIZENS

FUNDAMENTAL RIGHTS Visit: www.civilstap.com For Any Query Mail us: [email protected] or call us at - (+91)-8146207241 13 Subscribe to our YouTube Channel – CivilsTap by EduTap for free, quality and regular content.

. As citizens, we should abide by the Constitution, 1. This system ensures harmony between defend our country, promote harmony among all legislative and executive organs of the citizens, and protect the environment. Government. . However, the Constitution does not say anything 2. During British rule, Indians became familiar about enforcing these duties. with this form of Government. 3. It gives more credence to responsibility over stability. DIRECTIVE PRINCIPLES OF STATE 4. It provides scope for wide representation of POLICY diverse identities in the Government. 5. Executive authority is vested in a group of They are the guidelines incorporated in the individuals (council of ministers) and not in a Constitution to serve as the guiding force for policy single person. So, it prevents authoritarian formulations by state. tendencies. . Part IV of the Indian Constitution contains a 6. It prevents vacuum as there is always ready detailed chapter on Directive Principles. It lists alternate Government in case ruling power three things; loses majority. 1. Goals and objectives our society should adopt FEATURES OF PARLIAMENTARY FORM 2. Certain rights that Individual should enjoy Parliamentary system, also known as the other than fundamental rights. ‘Westminster’ model of Government, responsible 3. Certain policies that the government should government and cabinet Government is adopt characterised by following features. . Directive Principles include the right to . Presence of both nominal and real executives. . education, formation of panchayati raj Government is formed by that party which institutions all over the country, partial right to secures majority of votes in election for Lok work under employment guarantee programme Sabha.

etc. . Collective Responsibility of ministers to . Directive Principles resemble the ‘Instrument of parliament is the bedrock feature of this system. . Instructions’ enumerated in the Government of It provides for double membership for ministers India Act of 1935. of cabinet. They are part of executives as well as . These principles tries to establish ideals of social member of legislature.

and economic democracy. Their aim is to create . The role of Prime minister is very important. He “welfare state” in India. is the head of the Government and leads his cabinet. . Executive enjoys the right to get the legislature ADOPTING PARLIAMENTARY FORM dissolved in a parliamentary system. . Ministers hold the office till they enjoy OF GOVERNMENT confidence in the Parliament. In the Parliamentary form of Government the . There is homogeneity in political thoughts of executive is directly responsible to the legislature for party forming the Government. its policies and acts.

WHY PARLIAMENTARY FORM? . Constitution makers gave preference to British parliamentary system for Centre as well as states over American Presidential system due to following reasons;

Visit: www.civilstap.com For Any Query Mail us: [email protected] or call us at - (+91)-8146207241 14 Subscribe to our YouTube Channel – CivilsTap by EduTap for free, quality and regular content.

QUESTION 7 QUESTION 8 Q. In the context of India, which of the following Q. The main advantage of the parliamentary form principles is/are, implied institutionally in the of Governments is that (UPSC 2017) Parliamentary government? (UPSC 2013) (a) The executive and legislature work 1. Members of the Cabinet are Members of the independently. Parliament. (b) It provides continuity of policy and is more 2. Ministers hold the office till they enjoy efficient. confidence in the Parliament. (c) The executive remains responsible to the 3. Cabinet is headed by the Head of the State. legislature. Select the correct answer using the codes given (d) The head of the Government cannot be below: changed without election. (a) 1 and 2 only (b) 3 only Answer: C (c) 2 and 3 only (d) 1, 2 and 3 INTEGRATED AND INDEPENDENT Answer: A JUDICIARY DIFFERENCES WITH BRITISH SYSTEM Although Constitution makers preferred British The Indian Constitution establishes a judicial system System, they made some suitable changes in it as per that is integrated as well as independent. Indian scenario. These are: INTEGRATED JUDICIARY . While Britain has Monarchical system, India . The Constitution of India provides for a single follows Republican system in its place. So, our integrated judicial system. This means that head of state (President) is elected one not unlike some other federal countries of the world, hereditary (King or queen). India does not have separate State courts. . There is synthesis of Parliamentary Sovereignty . The structure of the judiciary in India is pyramidal and Judicial Supremacy in India. Britain follows with the Supreme Court at the top, High Courts the principle of parliamentary sovereignty only. below them and district and subordinate courts at the lowest level.

QUICK FACT INDEPENDENT JUDICIARY Principle of Judicial supremacy is an American concept that . Simply stated independence of judiciary means states that “courts are supreme over the other two branches of government and the Constitution”. that; 1. The other organs of the Government like the executive and legislature must not restrain or . In India head of Government (Prime Minister) interfere the functioning of the judiciary. can be a member of Lower house or Upper 2. Judges must be able to perform their house, however, in Britain he should be a functions without fear or favour. member of Lower house only. . Indian Constitution has ensured the . Principle of Legal responsibility is not followed independence of the judiciary through a number in India. Indian ministers did not need to of measures like countersign their official acts. 1. The legislature is not involved in the process of appointment of judges. 2. The judges have a fixed tenure. 3. Security of tenure ensures that judges could function without fear or favour.

Visit: www.civilstap.com For Any Query Mail us: [email protected] or call us at - (+91)-8146207241 15 Subscribe to our YouTube Channel – CivilsTap by EduTap for free, quality and regular content.

4. The judiciary is not financially dependent on SYSTEM OF LOCAL GOVERNMENT- either the executive or legislature. 5. The actions and decisions of the judges are PANCHAYATS AND MUNICIPALITIES immune from personal criticisms. Local Government is Government at the village and 6. Parliament cannot discuss the conduct of the district level. The organisation of local Governments judges except when the proceeding to is an important step in the direction of remove a judge is being carried out. democratisation.

BACKGROUND UNIVERSAL ADULT FRANCHISE . When the Constitution was prepared, the subject of local Government was assigned to the States. Adult franchise means that the right to vote should . Later it was felt that the subject of local be given to all adult citizens without the Government including panchayats did not discrimination of caste, class, colour, religion or sex. receive adequate importance in the CONSTITUTIONAL POSITION Constitution. . India adopted the principle of Universal adult . In this background, a three-tier Panchayati Raj franchise when the present Constitution was system of local Government was recommended enacted in 1949. for the rural areas. . The elections to the Lok Sabha and to the . But in many States those local bodies did not Legislative Assembly of every State shall be on have enough powers and functions to look after the basis of adult suffrage. the local development. . Every citizen of India who is not less than 18 73RD AND 74TH CONSTITUTIONAL AMENDMENT ACTS years of age has a right to vote. These Amendment Acts (1992) have given WHY ADULT FRANCHISE constitutional recognition to local Government It makes our democracy broad-based, enhances the which is not found in any other Constitution of the self-respect and prestige of the common people, and world. upholds the principle of equality and provides . 73rd amendment is about rural local protection to the minorities. Governments (Panchayati Raj Institutions). . 74th amendment made the provisions relating to SYSTEM OF SINGLE CITIZENSHIP urban local Government (Municipalities). . The 73rd and 74th Amendments came into force Indian Constitution provides for Single citizenship in 1993. only. There is no separate state citizenship. . The citizens in India owe allegiance only to the INDEPENDENT AGENCIES AS Union. . All citizens irrespective of the state to which they BULWARKS OF CONSTITUTION belong enjoy the same political and civil rights all Certain agencies are given constitutional recognition over the country. and exhaustive powers by Indian Constitution to . This is unlike USA, where a citizen is a citizen of work as bulwarks of Indian democracy. These are; USA and the state in which he/she resides. ELECTION COMMISSION . Autonomous authority responsible for QUICK FACT administering free and fair election processes in Article 9 of Indian Constitution-“No person shall be a citizen India at national and state level. of India or be deemed to be a citizen of India, if he has voluntarily acquired the citizenship of any foreign state”. . It administers elections to the Lok Sabha, Rajya Sabha, State Legislative Assemblies, State legislative Councils, and the offices of the President and Vice President of the country. Visit: www.civilstap.com For Any Query Mail us: [email protected] or call us at - (+91)-8146207241 16 Subscribe to our YouTube Channel – CivilsTap by EduTap for free, quality and regular content.

COMPTROLLER AND AUDITOR-GENERAL . It is an authority, established to audit all receipts and expenditure of the Government of India SCHEDULES OF INDIAN and the state Governments, including those of CONSTITUTION bodies and authorities substantially financed by the Government.

UNION PUBLIC SERVICE COMMISSION . It is India's premier central recruiting agency. . It is responsible for appointments to and examinations for All India services and group A & group B of Central services.

STATE PUBLIC SERVICE COMMISSION . State Public Service Commission (SPSC) are formed to carry out tasks of recruitment of suitable persons for various administrative vacancies under the state Government.

EMERGENCY PROVISIONS Emergency provisions are contained in Part XVIII of the Constitution, from Articles 352 to 360. It provides for three types of emergencies.

Emergency

National State Financial

External Armed President aggression rebellion Rule

. These provisions enable the Central Government to meet any abnormal situation effectively. . They serve the following purposes; 1. Safeguard the sovereignty, unity, integrity 2. Ensure security of the country 3. Preserve democratic political system, 4. Safeguard the Constitutional setup

Visit: www.civilstap.com For Any Query Mail us: [email protected] or call us at - (+91)-8146207241 17 Subscribe to our YouTube Channel – CivilsTap by EduTap for free, quality and regular content.

QUESTION 9 PREAMBLE OF INDIAN Q. Which one of the following Schedules of the Constitution of India contains provisions CONSTITUTION regarding Anti-defection Act? (UPSC 1998 ; 2014) (a) Second Schedule The Preamble of the Indian Constitution serves as a (b) Fifth Schedule brief introductory statement that sets out the (c) Eight Schedule guiding purpose, principles and philosophy of the (d) Tenth Schedule Indian Constitution. The inspiration and the motto of the citizens are explained in it. Answer: D . It provides an insight about the mind of constitution makers. IMPORTANT FACTS QUESTION 10 . The Preamble of the Indian Constitution is Q. Which one of the following statements primarily based on the ‘Objective Resolution’ correctly describes the Fourth Schedule of the written by Jawaharlal Nehru. Constitution of India? (UPSC 2001 & 2004) . It has been amended only once by the 42nd (a) It lists the distribution of powers between the Constitutional Amendment Act (1976), which Union and the states. added three new words—socialist, secular and (b) It contains the languages listed in the integrity. Constitution. . It is an integral part of our Constitution. (c) It contains the provisions relating to the . It is neither a source of power to legislature nor administration of tribal areas. a prohibition upon the powers of legislature. (d) It allocates seats in the Council of States (Rajya . It is non-justiciable in nature. Sabha) PREAMBLE IN PRESENT FORM Answer: D

“We, THE PEOPLE OF INDIA, having QUESTION 11 Q. Which one of the following schedules of the solemnly resolved to constitute India into a Indian Constitution lists the names of States and SOVEREIGN SOCIALIST SECULAR specifies their territories? (UPSC 2003) DEMOCRATIC REPUBLIC and to secure to (a) First all its citizens: (b) Second JUSTICE, social, economic and political;

(c) Third LIBERTY of thought, expression, belief, (d) Fourth faith and worship; Answer: A EQUALITY of status and of opportunity; and to promote among them among all; FRATERNITY assuring the dignity of the individual and the unity and integrity of the Nation;

IN OUR CONSTITUENT ASSEMBLY this twenty-sixth day of November, 1949, do HEREBY ADOPT, ENACT AND GIVE TO OURSELVES THIS CONSTITUTION”. Visit: www.civilstap.com For Any Query Mail us: [email protected] or call us at - (+91)-8146207241 18 Subscribe to our YouTube Channel – CivilsTap by EduTap for free, quality and regular content.

COMPONENTS OF PREAMBLE . Positively all persons will be equally entitled to the freedom of conscience and the right freely Preamble of Indian Constitution provides insights to profess, practice and propagate the religion of about following matters. their choice.

SOURCE OF ITS AUTHORITY DEMOCRACY . It indicates that the Constitution has established . The phrase “We, THE PEOPLE OF INDIA” a form of Government which gets its authority emphasises that the Constitution is made by and from the will of the people. for the Indian people and not given to them by . The rulers are elected by the people and are any outside power. responsible to them. . Constitution derives its authority from the . Indian Constitution provides for representative people of India. parliamentary democracy under which the . It also emphasizes the concept of popular executive is responsible to the legislature for all sovereignty. its policies and actions. . The word 'democratic' not only refers to political NATURE OF INDIAN STATE but also to social & economic democracy.

It states the nature of the Indian State, which is REPUBLIC Sovereign, Socialist, Secular, Democratic and . In a republican form of Government, the head of Republic. state is elected and not a hereditary monarch. . Thus, India has a President who is elected and SOVEREIGN has a fixed term of office. . Sovereignty means the independent authority of . There's an absence of any privileged class and a State. It has two aspects- external and internal. hence all public offices being opened to every . External sovereignty of India means that it can citizen without any discrimination. acquire foreign territory and also cede any part of the Indian Territory, subject to limitations DATE OF THE ADOPTION OF INDIAN imposed by the Constitution. CONSTITUTION . Internal sovereignty refers to the relationship between the states and the individuals within its “Twenty-sixth day of November, 1949” is territory. mentioned in the Preamble as the date on which the Constituent Assembly adopted, enacted and gave to SOCIALIST themselves this Constitution. . Certain Directive Principles of State Policy have . Some provisions of the Constitution came into socialist content in them. force on November 26, 1949 itself. . The term was added in the Indian constitution by . January 26, 1950 is referred to in the the 42nd Amendment in 1976. Constitution as the ‘date of its commencement’, . Socialism as a social philosophy stresses more on and celebrated as the Republic Day. societal equality. Under it, means of production and distribution are owned by the State. OBJECTIVES OF THE INDIAN . India adopted Mixed Economy, where apart CONSTITUTION from the state, there will be private production too. JUSTICE Justice among the citizens is necessary to maintain SECULAR order in society. . By the 42nd Amendment in 1976, the term . The objective of justice—social, economic and "Secular" was also incorporated in the Preamble. political—has been taken from the Russian . It means that the Indian state will have no Revolution (1917). religion of its own. Visit: www.civilstap.com For Any Query Mail us: [email protected] or call us at - (+91)-8146207241 19 Subscribe to our YouTube Channel – CivilsTap by EduTap for free, quality and regular content.

. Justice is promised through various provisions of . This refers to a feeling of brotherhood and a Fundamental Rights and Directive Principles of sense of belonging among the citizens. State Policy provided by the Constitution of . It embraces psychological as well as territorial India. dimensions of National Integration. . The ideals of liberty, equality and fraternity in . Constitution promotes this feeling of fraternity our Preamble have been taken from the French by the system of single citizenship and Revolution. fundamental duties. . Unity and Integrity embraces psychological as well as territorial dimensions of National • Absence of privileges Integration. Social Justice • No social discrimination

QUESTION 12 Q. Which one of the following objectives is not • Equitable distribution of embodied in the Preamble to the Constitution of Economic wealth Justice India? (UPSC 2017) • Elimination of inequalities (a) Liberty of thought (b) Economic liberty (c) Liberty of expression • Right to vote Political Justice (d) Liberty of belief • Right to contest in elections Answer: B

EQUALITY

It means removing all types of discriminations from society to build a healthy environment for the QUESTION 13 people to live in. Everyone is equal before the law. Q. Which one of the following reflects the most . Preamble secures to all citizens appropriate relationship between law and 1. Equality of status liberty? (UPSC 2018) 2. Equality of opportunity (a) If there are more laws, there is less liberty. . This provision embraces three dimensions of (b) If there are no laws, there is no liberty. equality—civic, political and economic. (c) If there is liberty, laws have to be made by the people. LIBERTY (d) If laws are changed too often, liberty is in . The term ‘liberty’ signifies; danger. 1. The quality or state of being free.

2. Freedom from arbitrary or despotic rule. Answer: B 3. Positive enjoyment of various social, political, or economic rights and privileges. IMPORTANT JUDGEMENTS RELATED . Preamble secures to all citizens of India liberty TO PREAMBLE through their Fundamental Rights. . Liberty conceived by the Preamble or Initially there was two main confusion about the fundamental rights is not absolute but qualified. Preamble; . Liberty entails the responsible use of freedom . It is a part of the Constitution or not. under the rule of law without depriving anyone . It can be amended or not. else of their freedom. It was resolved through the following judgements.

FRATERNITY

Visit: www.civilstap.com For Any Query Mail us: [email protected] or call us at - (+91)-8146207241 20 Subscribe to our YouTube Channel – CivilsTap by EduTap for free, quality and regular content.

AMENDMENT OF INDIAN 1960 Berubari UnionCase CONSTITUTION •Supreme Court specifically opined that Preamble is not a part of the Constitution. The Constitution of India was adopted on 26 November 1949. Its implementation formally Kesavnanda Bharati started from 26 January 1950. Since then, the same 1973 Constitution continues to function as the framework Case within which the Government of our country • Supreme Court rejected the earlier opinion and held operates. It is due to the fact that that Preamble is a part of the Constitution. . Our Constitution accepts the necessity of • Preamble can be amended, subject to the condition modifications according to changing needs of the that no amendment is done to the ‘basic features’. society. . The Constitution can be amended if so required. However it is protected from unnecessary and 1995 frequent changes. LIC of India Case . The procedure laid down for its amendment is •Supreme Court again held that the Preamble is an neither as easy as in Britain nor as difficult as in integral part of the Constitution. USA. . Sovereignty of elected representatives PRESENT POSITION OF PREAMBLE (parliamentary sovereignty) is the basis of the . Preamble is a part of the Constitution. amendment procedure. . Preamble can be amended, subject to the condition that no amendment is done to the TYPES OF AMENDMENTS ‘basic features’. Different ways of amending the Constitution by the Parliament are shown below. QUESTION 14 Q. The mind of the makers of the Constitution of India is reflected in which of the following? (a) Preamble (UPSC 2017) (b) Fundamental Rights Types of (c) Directive Principles of State Policy Amendment (d) Fundamental Duties

Answer: A Through Through Article Ordinary 368 Legislation

Special Special majority Simple majority in + Ratified by majority in Parliament Legislatures of Parliament half the states

Visit: www.civilstap.com For Any Query Mail us: [email protected] or call us at - (+91)-8146207241 21 Subscribe to our YouTube Channel – CivilsTap by EduTap for free, quality and regular content.

. Article 368 in Part XX of the Constitution deals PROCEDURE FOR AMENDMENT OF with the powers of Parliament to amend the Constitution and its procedure. INDIAN CONSTITUTION

INTRODUCTION OF BILL ARTICLE 368 OF INDIAN CONSTITUTION . An amendment of the Constitution may be

“Notwithstanding anything in this Constitution, Parliament initiated only by the introduction of a Bill for the may in exercise of its constituent power amend by way of purpose in either House of Parliament and not in addition, variation or repeal any provision of this state legislatures. Constitution in accordance with the procedure laid down in this article” . The Bill for the amendment can be introduced by a minister or by a private member.

. There is no need for prior permission of President for introducing the Bill.

PASSING OF THE BILL By Special . Each House must pass the Bill separately. Joint By Simple By Special Majority sitting is not provided for Constitutional Amendment Bills. Majority Majority and Consent . Both Houses must pass the Amendment Bill with of States special majorities separately. Second, Fifth and Sixth Schedule Manner of Fundamental Election of SPECIAL MAJORITY OF HOUSE REQUIRES Rights the President Citizenship 1. Those voting in favour of the Amendment Bill Matters should constitute at least half of the total strength of that House and Any of the 2. The supporters of the Amendment Bill must also Elections to Directive lists in the Parliament Seventh constitute two-thirds of those who actually take Principles of and state Schedule part in voting. legislatures State Policy ROLE OF STATES Matters of Supreme . When an amendment aims to modify an article Union Other Court and Territories provisions high courts related to federal provisions of the Constitution, not covered Legislatures of half the States have to pass the Admission or by the first amendment bill by simple majority. establishment and third Extent of the of new states categories executive PRESIDENTIAL APPROVAL power of the Delimitation Union and . An Amendment Bill, like all other bills, goes to matters the states the President for his assent, but in this case, the President has no powers to send it back for Salaries and Representation reconsideration. allowances of of states in After the President’s assent, the bill becomes an Act the members Parliament of Parliament and the Constitution stands amended in accordance with the terms of the Act. Rules of Distribution procedure in of legislative Parliament powers

Use of official Article 368 language itself

Visit: www.civilstap.com For Any Query Mail us: [email protected] or call us at - (+91)-8146207241 22 Subscribe to our YouTube Channel – CivilsTap by EduTap for free, quality and regular content.

. There is no provision for a special body for QUESTION 15 amending the Constitution. Q. Which of the following are matters on which a . State legislatures cannot initiate any bill or constitutional amendment is possible only with proposal for amending the Constitution the ratification of the legislatures of not less than one-half of the states? (UPSC 1995) NO TIME LIMIT 1. Election of the President. . There is no prescribed time frame within which 2. Representation of the States in Parliament. the state legislatures should ratify or reject an 3. Any of the Lists in the 7th schedule. amendment related to federal provisions. 4. Abolition of the Legislative Council of a State. DEADLOCK UNSETTLED

. In case of deadlock, there is no provision of joint (a) 1, 2 and 3 sitting to resolve the matter. (b) 1, 2 and 4 (c) 1, 3 and 4 FREQUENCY OF AMENDMENTS (d) 2, 3 and 4 . In past 70 years Indian Constitution has been amended 104 times (as on January 2020). Answer: A . Given the relatively difficult method of amending the Constitution, the number of amendments appears quite high. QUESTION 16 Q. Consider the following statements 1. An amendment to the Constitution of India can be initiated by an introduction of a bill in the Lok Sabha only. 2. If such an amendment seeks to make changes in the federal character of the Constitution, the amendment also requires to be ratified by the legislature of all the States of India. Which of the statement(s) given above is/are correct? (UPSC 2003) (a) Only 1 (b) Only 2 (c) Both 1 and 2 (d) Neither 1 nor 2

Answer: D Figure 1- Amendments per decade

However, barring the first decade after the ISSUES WITH THE AMENDMENT commencement of the Constitution, every decade has witnessed a steady stream of amendments. This PROCEDURE means that irrespective of the nature of politics and the party in power, amendments were required to Critics have identified following issues in the current be made from time to time. amendment procedure;

EXCLUSIVE POWER TO PARLIAMENT CONTENT OF AMENDMENTS MADE . Major part of the Constitution can be amended SO FAR by the Parliament alone either by a special majority or by a simple majority. Amendments made so far in the Indian Constitution may be classified in three groups. Visit: www.civilstap.com For Any Query Mail us: [email protected] or call us at - (+91)-8146207241 23 Subscribe to our YouTube Channel – CivilsTap by EduTap for free, quality and regular content.

AMENDMENTS OF TECHNICAL OR ADMINISTRATIVE Parliament's authority to amend the Constitution, NATURE particularly the chapter on the fundamental rights of They are amendments only in the legal sense, but in citizens, was challenged as early as in 1951. matter of fact, they made no substantial difference to the provisions. SHANKARI PRASAD CASE (1951) . Amendment that increased the age of . The First Constitution Amendment Act, 1951 retirement of High Court judges from 60 to 62 was challenged in the Shankari Prasad case. years falls in this category. . The Amendment was challenged on the ground that it violates the Part-III of the Constitution and AMENDMENTS TO DEAL WITH DIFFERING therefore, should be considered invalid. INTERPRETATIONS Various amendments are a product of different JUDICIAL RESPONSE interpretations of the provisions in the Constitution . Supreme Court held that the Parliament, under given by the judiciary and the Government of the Article 368, has the power to amend any part of day. the Constitution. . Parliament had to insert an amendment . Parliament can abridge or take away any of the underlining one particular interpretation as the Fundamental Rights by enacting a Constitutional authentic one. Amendment Act. . One of the matter was the relationship between SAJJAN SINGH CASE (1965) fundamental rights and directive principles. . Supreme Court gave the same ruling as . In the period 1970-1975, the Parliament pronounced in Shankari Prasad Case. repeatedly made amendments to overcome the adverse interpretations by the judiciary. GOLAKNATH CASE (1967) . In that case, the constitutional validity of the AMENDMENTS THROUGH POLITICAL CONSENSUS Seventeenth Amendment Act (1964), which These amendments have been made as a result of inserted certain state acts in the Ninth Schedule, the consensus among the political parties. was challenged. . This changes had to be made in order to reflect the prevailing political philosophy and JUDICIAL RESPONSE aspirations of the society. . In this case the Supreme Court overruled its . Many of the amendments of the post-1984 earlier decision. period are instances of this trend. . The Supreme Court held that the Parliament has . These amendments include the 61st no power to amend Part III of the constitution as amendment bringing down the minimum age the fundamental rights are transcendental and for voting from 21 to 18 years, the 73rd and the immutable. 74th amendments, etc. . Article 368 only lays down the procedure to amend the constitution and does not give DOCTRINE OF BASIC STRUCTURE OF absolute powers to the Parliament to amend any THE INDIAN CONSTITUTION part of the Constitution. PARLIAMENT REACTION One thing that has had a long lasting effect on the . Parliament reacted to judicial ruling by passing evolution of the Indian Constitution is the theory of the 24th Constitutional Amendment Act. the basic structure of the Constitution. . It gave absolute power to the Parliament to . There is no mention of this theory in the make any changes in the constitution including Constitution. It has emerged from judicial the fundamental rights. interpretation. KESAVANANDA BHARATI CASE (1973) EMERGENCE OF THIS DOCTRINE . In Kesavananda Bharti case, the Supreme Court upheld the validity of the 24th Constitution Visit: www.civilstap.com For Any Query Mail us: [email protected] or call us at - (+91)-8146207241 24 Subscribe to our YouTube Channel – CivilsTap by EduTap for free, quality and regular content.

Amendment Act by reviewing its decision in . This doctrine helps to protect and preserve the Golaknath case. spirit of the Constitution document. . The Supreme Court held that the Parliament has . The idea is to preserve the nature of Indian power to amend any provision of the democracy and protect the rights and liberties of constitution. people. . At the same time, it laid down a new doctrine of the ‘basic structure’ of the Constitution. ELEMENTS OF BASIC STRUCTURE . There is no mention of the term “Basic Structure”

anywhere in the Indian Constitution. SIGNIFICANCE OF KESAVANANDA BHARATI CASE . The basic structure of the Constitution has not RULING been explicitly defined by the Judiciary. 1. It has set specific limits to Parliament’s power to amend . The Kesavananda Bharti Case judgement (1973) the Constitution. It says that no amendment can violate listed some basic structures of the constitution the basic structure of the Constitution; 2. It allows Parliament to amend any and all parts of the as: Constitution (within this limitation); and 1. Supremacy of the Constitution 3. It places the Judiciary as the final authority in deciding if 2. Unity and sovereignty of India an amendment violates basic structure and what 3. Democratic and republican form of constitutes the basic structure. government 4. Federal character of the Constitution 5. Secular character of the Constitution INDIRA NEHRU GANDHI CASE (1975) 6. Separation of power . The Supreme Court reaffirmed its concept of 7. Individual freedom basic structure. . Over time various judgements, led to the PARLIAMENT REACTION TO THIS DOCTRINE emergence of following as ‘basic features’ of the . Parliament reacted to this judicially innovated Constitution doctrine by enacting the 42nd Amendment Act 1. Rule of law (1976). 2. Judicial review . This Act amended Article 368 and declared that 3. Parliamentary system there is no limitation on the constituent power 4. Rule of equality of Parliament and no amendment can be 5. Harmony and balance between the questioned in any court on any ground. Fundamental Rights and DPSP 6. Free and fair elections MINERVA MILLS CASE (1980) 7. Limited power of the parliament to amend . The concept of basic structure was reaffirmed by the Constitution under Article 368. adding 'judicial review' and the 'balance 8. Power of the Supreme Court under Articles between Fundamental Rights and Directive 32, 136, 142 and 147. Principles' to the basic features. 9. Power of the High Court under Articles 226 . It invalidated the above changes made in Article and 227. nd 368 by 42 Amendment Act. Thus the doctrine of the basic structure helps to WHAT IS THE DOCTRINE OF BASIC STRUCTURE? prevent Executive and legislative excesses. This is required as a shield against an all-powerful “It states that the Constitution of India has certain Parliament, which can resort to an overuse of Article basic features that cannot be altered or destroyed 368. through amendments by the Parliament”. . The basic structure doctrine has consolidated the balance between rigidity and flexibility by saying that certain parts cannot be amended. Visit: www.civilstap.com For Any Query Mail us: [email protected] or call us at - (+91)-8146207241 25 Subscribe to our YouTube Channel – CivilsTap by EduTap for free, quality and regular content.

1. It provided for two dominion states: India and MCQS FOR PRACTICE Pakistan. 2. It provided, for each dominion, a governor- Q1. Consider the following pairs and identify the general, who was to be appointed by the ones which are correctly matched: Constituent assembly of respective dominion. Features Introducing Act 3. Those civil servants who had been appointed 1. Local representation in Indian Council Act of before the August 15, 1947, will continue in legislative council 1861 service with same privileges 2. Indian representative Charter Act of 1853 Which of the statements given above is/are in legislative council correct? 3. Principle of separate Indian Council Act of (a) 1 only electorate 1909. (b) 1 and 3 only Select the correct answer using the codes given (c) 2 and 3 only below: (d) All of the above (a) 1 and 2 only (b) 2 and 3 only Q5. Which of the following was the first Indian to (c) 1 and 3 only join viceroy’s executive council? (d) 3 only (a) Bal Gangadhar Tilak (b) Satyendra Prasad Sinha Q2. Examine the following statements with regard (c) Gopal Krishna Gokhale to the Indian Council Act of 1909- (d) Mahatma Gandhi 1. It established Indian dominance in the provincial, but not central, legislative bodies. Q6. In the context of recruitment of civil servants 2. The members were given the right to move during British rule in India, the open competition resolutions on matters of general public interest. for selection and recruitment of civil servants was Which of the above statements is/are correct? started by: (a) 1 only (a) Charter Act of 1833 (b) 2 only (b) Charter Act of 1853 (c) Both 1 and 2 (c) Indian Council Act of 1861 (d) Neither 1 nor 2 (d) Government of India Act 1858

Q3. Which of the following statements in context of Q7. In relation to various developments during Indian Council Act of 1861 correct? British rule, arrange the following events in correct 1. The ‘Portfolio’ System was introduced by chronological order: Viceroy Lord Canning. (a) Indian Council Act of 1861 2. It allowed viceroy to issue ordinances if (b) Act of good Government for India legislative council not in session. (c) Act of Settlement Select the correct answer using the codes given Select the correct answer using the codes given below: below: (a) 1 only (a) 2-3-1 (b) 2 only (b) 1-3-2 (c) Both 1 and 2 (c) 3-1-2 (d) Neither 1 nor 2 (d) 3-2-1

Q4. Consider the following statements in context of Q8. Consider the following statements in context India Independence Act of 1947: of Preamble of Constitution of India: 1. Preamble is a part of the Constitution.

Visit: www.civilstap.com For Any Query Mail us: [email protected] or call us at - (+91)-8146207241 26 Subscribe to our YouTube Channel – CivilsTap by EduTap for free, quality and regular content.

2. Preamble cannot be amended by parliament. Q12. Who among the following was/were not the Which of the statements given above is/are head of any of the major committees of constituent correct? assembly? (a) 1 only 1. Sardar Patel (b) 2 only 2. Hansa Mehta (c) Both 1 and 2 3. Jawaharlal Nehru (d) Neither 1 nor 2 Select the correct answer using the codes given below: Q9. Consider the following pairs and identify the (a) 1 and 2 only ones which are correctly matched: (b) 2 only Features Borrowed from (c) 1 and 3 only 1. Parliamentary British Constitution (d) 2 and 3 only Sovereignty 2. Judicial Supremacy French Constitution Q13. With reference to Constituent Assembly of 3. Concept of Justice USSR Constitution India, Which of the following statement is not Select the correct answer using the codes given correct? below: (a) It included fifteen women. (a) 1 and 2 only (b) Mahatma Gandhi was president of constituent (b) 2 and 3 only assembly till 30 January, 1948. (c) 1 and 3 only (c) Drafting committee of the constituent assembly (d) 3 only was headed by Dr. B.R.Ambedkar. (d) Minor committees were headed by both Q10. Which of the following statements is/are assembly members and non-assembly members. correct? 1. Indian Constitution provides for Single Q14. Which of the following statements is/are not citizenship only. correct? 2. Part III of the Indian Constitution contains a 1. The term ‘federation’ has nowhere been used in detailed chapter on Directive Principles. the Constitution. Select the correct answer using the codes given 2. All India services violate the principle of below: federalism under the Constitution. (a) 1 only Select the correct answer using the codes given (b) 2 only below: (c) Both 1 and 2 (a) 1 only (d) Neither 1 nor 2 (b) 2 only (c) Both 1 and 2 Q11. Which of the following is/are features of (d) Neither 1 nor 2 Parliamentary form of Government? 1. Collective Responsibility Q15. Modified version of Objective resolution 2. Presence of both nominal and real executives. forms the 3. It provides for double membership for ministers. (a) Preamble. 4. System of Majority party rule (b) Fundamental Rights Select the correct answer using the codes given (c) Directive Principles of State policy below: (d) Fundamental Duties (a) 1, 2, 3 and 4 (b) 2 only (c) 1 and 2 only (d) None of the above

Visit: www.civilstap.com For Any Query Mail us: [email protected] or call us at - (+91)-8146207241 27 Subscribe to our YouTube Channel – CivilsTap by EduTap for free, quality and regular content.

Q16. Which of the following is/are features 1. The Constitution was adopted on November 26, adopted by Indian Constitution from American 1949. Constitution? 2. Constitution of India came into force on 26 1. Federation with a strong Centre November, 1950. 2. Fundamental rights 3. Dr. Rajendra Prasad is recognised as the ‘Father 3. Independence of judiciary of the Constitution of India’ 4. Advisory jurisdiction of the Supreme Court Select the correct answer using the codes given 5. Procedure established by law below: Select the correct answer using the codes given (a) 1 and 2 only below: (b) 2 and 3 only (a) 3 and 4 only (c) 1 only (b) 2 only (d) None of the above (c) 2 and 3 only (d) All of the above Q21. Preamble of Indian constitution provides insights about following matters Q17. Which among the following was added to the (a) Objectives of Indian Constitution as liberty, preamble by 42nd Amendment to the Constitution? equality and fraternity only. (a) Liberty of thought and expression (b) Nature of Indian State as Sovereign Socialist (b) Socialist, Secular and Integrity Secular Democratic Republic. (c) Equality of opportunity (c) Date of commencement of the Indian (d) All of the above Constitution. (d) All of the above Q18. The Government of India Act of 1935 provided for Q22. Which of the following is/are features of (a) Establishment of an All-India Federation Indian Constitution? consisting of provinces and princely States as 1. Federation with a strong Centre. units. 2. System of dual citizenship. (b) Separate chapter on Fundamental Rights for 3. Integrated as well as independent judicial Indians. system. (c) Diarchy was introduced in the provinces. 4. Lengthiest constitution of the World at present. (d) It abolished the Council of India, established by Select the correct answer using the codes given the Government of India Act of 1858. below: (a) 3 and 4 only Q19. Consider the following statements: (b) 2 only 1. Various Indian states had come into an (c) 1, 3 and 4 only agreement to form the Indian federation. (d) All of the above 2. States have right to secede from the federation. Which of the statements given above is/are Q23. Which of the following statements is/are not correct? correct? (a) 1 only 1. From the British side, Cripps Mission (March, (b) 2 only 1942) made the first in principle approval of the (c) Both 1 and 2 constituent assembly for India. (d) Neither 1 nor 2 2. Constituent Assembly of India was partly elected and partly nominated body. Q20. Consider the following statements in the Select the correct answer using the codes given context of Constitution of India: below: (a) 1 only

Visit: www.civilstap.com For Any Query Mail us: [email protected] or call us at - (+91)-8146207241 28 Subscribe to our YouTube Channel – CivilsTap by EduTap for free, quality and regular content.

(b) 2 only 2. There is synthesis of Parliamentary Sovereignty (c) Both 1 and 2 and Judicial Supremacy in India. Britain follows (d) Neither 1 nor 2 the principle of parliamentary sovereignty only. Select the correct answer using the codes given Q24. Consider the following pairs of adoption date below: of various symbol of Indian State and identify the (a) 1 only ones which are correctly matched: (b) 2 only 1. National Flag - July 22, 1947 (c) Both 1 and 2 2. National Anthem - January 24, 1950 (d) Neither 1 nor 2 3. National Song - January 26, 1950 Select the correct answer using the codes given Q28. Which among the following is not a factor below: responsible for the bulkiness of Indian (a) 1 only Constitution? (b) 2 and 3 only (a) Vast size and large diversity of India (c) 1 and 2 only (b) There is single Constitution for Centre as well as (d) 3 only States (c) Due to dominance of lawyers in constituent Q25. Which of the following statements is/are not assembly correct? (d) Legacy of Government of India Act of 1919. 1. Constituent Assembly also worked as the provisional Parliament of India with Dr. Rajendra Q29. Consider the following statements: Prasad as its elected speaker. 1. 73rd Constitutional Amendment is about rural 2. Muslim League attended the first meeting of local Governments (Panchayati Raj Institutions). Constituent assembly held on December 9, 1946. 2. 74th Constitutional Amendment made the Select the correct answer using the codes given provisions relating to urban local Government below: (Municipalities). (a) 1 only Which of the statements given above is/are (b) 2 only correct? (c) Both 1 and 2 (a) 1 only (d) Neither 1 nor 2 (b) 2 only (c) Both 1 and 2 Q26. Original Constitution of India does not (d) Neither 1 nor 2 provided for (a) Preamble Q30. Which of the following statements is/are not (b) Fundamental Rights correct? (c) Directive Principles of State policy 1. Sovereignty means the independent authority of (d) Fundamental Duties a state. It has two aspects- external and internal. 2. The word 'democratic' in Indian Preamble only Q27. Which of the following is/are the differences refers to social & economic democracy only. between the Indian and British Parliamentary Select the correct answer using the codes given system of Government? below: 1. In India head of Government (Prime Minister) (a) 1 only can be a member of Lower house or Upper (b) 2 only house, however, in Britain he should be a (c) Both 1 and 2 member of Lower house only. (d) Neither 1 nor 2

Visit: www.civilstap.com For Any Query Mail us: [email protected] or call us at - (+91)-8146207241 29 Subscribe to our YouTube Channel – CivilsTap by EduTap for free, quality and regular content.

2. It considerably increased the size of the ANSWERS & EXPLANATIONS legislative councils, both Central and provincial. Q1. Consider the following pairs and identify the 3. It allowed Non official majority (Indian ones which are correctly matched: dominance) in the provincial, but not Features Introducing Act central, legislative bodies. 1. Local representation in Indian Council Act of 4. Members were also enabled to debate legislative council 1861 budgetary and complementary matters and 2. Indian representative Charter Act of 1853 table resolutions on matters of general in legislative council interest 3. Principle of separate Indian Council Act of electorate 1909. Q3. Which of the following statements in context of Select the correct answer using the codes given Indian Council Act of 1861 correct? below: 1. The ‘Portfolio’ System was introduced by (a) 1 and 2 only Viceroy Lord Canning. (b) 2 and 3 only 2. It allowed viceroy to issue ordinances if (c) 1 and 3 only legislative council not in session. (d) 3 only Select the correct answer using the codes given Answer: D below: Explanation: (a) 1 only . Local representation was introduced in the new (b) 2 only Legislative council created under Charter Act of (c) Both 1 and 2 1853. (d) Neither 1 nor 2 . The beginning of representative Institutions in Answer: C India can be traced to the Indian Council Act of Explanation: 1861. . Indian Council Act of 1861 provided for some . Indian Council Act of 1909 provides for the new tools of administration. separate electorate for Muslims giving birth to . The ‘Portfolio’ System was introduced by Viceroy communal representation in India. Lord Canning. . The Viceroy was allowed to issue ordinances Q2. Examine the following statements with regard lasting six months if the Legislative Council was to the Indian Council Act of 1909- not in session. 1. It established Indian dominance in the provincial, but not central, legislative bodies. Q4. Consider the following statements in context of 2. The members were given the right to move India Independence Act of 1947: resolutions on matters of general public interest. 1. It provided for two dominion states: India and Which of the above statements is/are correct? Pakistan. (a) 1 only 2. It provided, for each dominion, a Governor- (b) 2 only General, who was to be appointed by the (c) Both 1 and 2 Constituent assembly of respective dominion. (d) Neither 1 nor 2 3. Those civil servants who had been appointed Answer: C before the August 15, 1947, will continue in Explanation: service with same privileges . Indian Council Act of 1909 made the following Which of the statements given above is/are changes: correct? 1. It made changes in Legislative Councils in (a) 1 only composition as well as powers. (b) 1 and 3 only

Visit: www.civilstap.com For Any Query Mail us: [email protected] or call us at - (+91)-8146207241 30 Subscribe to our YouTube Channel – CivilsTap by EduTap for free, quality and regular content.

(c) 2 and 3 only Q7. In relation to various developments during (d) All of the above British rule, arrange the following events in correct Answer: B chronological order: Explanation: (a) Indian Council Act of 1861 Important features of the Indian Independence Act (b) Act of good Government for India of 1947 are: (c) Act of Settlement . It provided for the partition of India and creation Select the correct answer using the codes given of two independent dominions of India and below: Pakistan. (a) 2-3-1 . It abolished the office of viceroy and provided, (b) 1-3-2 for each dominion, a governor-general, who was (c) 3-1-2 to be appointed by the British King on the advice (d) 3-2-1 of the dominion cabinet. Answer: D . The members of the civil services appointed Explanation: before August 15, 1947 would continue to enjoy Government of India Act of 1858 was also known as all benefits that they were entitled to till that the Act for good Government of India. time. . To rectify the defects of the Regulating Act of 1773, the British Parliament passed the Q5. Which of the following was the first Indian to Amending Act of 1781, which was also known as join viceroy’s executive council? the Act of Settlement. (a) Bal Gangadhar Tilak (b) Satyendra Prasad Sinha Q8. Consider the following statements in context (c) Gopal Krishna Gokhale of Preamble of Constitution of India: (d) Mahatma Gandhi 1. Preamble is a part of the Constitution. Answer: B 2. Preamble cannot be amended by the Parliament. Explanation: Which of the statements given above is/are . Satyendra Prasad Sinha became the first Indian correct? to join the Viceroy’s Executive Council. He was (a) 1 only appointed as the law member. (b) 2 only (c) Both 1 and 2 Q6. In the context of recruitment of civil servants (d) Neither 1 nor 2 during British rule in India, the open competition Answer: A for selection and recruitment of civil servants was Explanation: started by: . The Supreme Court through its various (a) Charter Act of 1833 judgements held that the Preamble is a part of (b) Charter Act of 1853 the Constitution. (c) Indian Council Act 0f 1861 . The Court also clarified that the Preamble can be (d) Government of India Act 1858 amended, subject to the condition that no Answer: B amendment is done to the ‘basic features’ of Explanation: Indian Constitution. . Charter Act of 1853 introduced the open competition for selection and recruitment of civil servants. The higher posts in civil service was thus thrown open to the Indians also.

Visit: www.civilstap.com For Any Query Mail us: [email protected] or call us at - (+91)-8146207241 31 Subscribe to our YouTube Channel – CivilsTap by EduTap for free, quality and regular content.

Q9. Consider the following pairs and identify the 4. System of Majority party rule ones which are correctly matched: Select the correct answer using the codes given Features Borrowed from below: 1. Parliamentary British Constitution (a) 1, 2, 3 and 4 Sovereignty (b) 2 only 2. Judicial Supremacy French Constitution (c) 1 and 2 only 3. Concept of Justice USSR Constitution (d) None of the above Select the correct answer using the codes given Answer: A below: Explanation: (a) 1 and 2 only In the Parliamentary form of Government the (b) 2 and 3 only executive is directly responsible to the legislature for (c) 1 and 3 only its policies and acts. (d) 3 only . Features of Parliamentary System Answer: C 1. Collective Responsibility of ministers to Explanation: parliament. The concept of Parliamentary Sovereignty in Indian 2. Presence of both nominal and real executives. Constitution is taken from British Constitution. 3. The ministers are members of both the . Concept of Judicial supremacy is an important legislature and the executive (Double feature of American Constitution. membership). . The ideal of Justice was borrowed from 4. The political party which secures majority seats Constitution of erstwhile USSR. in the Lok Sabha forms the government. 5. The role of Prime minister is very important. He Q10. Which of the following statements is/are is the head of the government. correct? 6. There is homogeneity in political thoughts of 1. Indian constitution provides for Single citizenship party forming the government. only. 2. Part III of the Indian Constitution contains a Q12. Who among the following was/were not the detailed chapter on Directive Principles. head of any of the major committees of constituent Select the correct answer using the codes given assembly? below: 1. Sardar Patel (a) 1 only 2. Hansa Mehta (b) 2 only 3. Jawaharlal Nehru (c) Both 1 and 2 Select the correct answer using the codes given (d) Neither 1 nor 2 below: Answer: A (a) 1 and 2 only Explanation: (b) 2 only Although the Indian Constitution is federal and (c) 1 and 3 only envisages a dual polity it provides for only a single (d) 2 and 3 only citizenship, that is, the Indian citizenship. Answer: B . Part III of the Indian Constitution contains a Explanation: detailed chapter on Fundamental Rights. In total there were eight major committees of the Constituent Assembly. None of the major Q11. Which of the following is/are features of Committees were headed by Women (Hansa Parliamentary form of Government? Mehta). 1. Collective Responsibility 2. Presence of both nominal and real executives. 3. It provides for double membership for ministers.

Visit: www.civilstap.com For Any Query Mail us: [email protected] or call us at - (+91)-8146207241 32 Subscribe to our YouTube Channel – CivilsTap by EduTap for free, quality and regular content.

Q13. With reference to Constituent Assembly of The Preamble to the Indian Constitution is modified India, Which of the following statement is not form of the ‘Objectives Resolution’, drafted and correct? moved by Jawaharlal Nehru, and adopted by the (a) It included fifteen women. Constituent Assembly. (b) Mahatma Gandhi was president of constituent assembly till 30 January, 1948. (c) Drafting committee of the constituent assembly Q16. Which of the following is/are features was headed by Dr. B.R.Ambedkar. adopted by Indian Constitution from American (d) Minor committees were headed by both Constitution? assembly members and non-assembly members. 1. Federation with a strong Centre Answer: B 2. Fundamental rights Explanation: Mahatma Gandhi was not the member 3. Independence of judiciary of Constituent Assembly. He never presided the 4. Advisory jurisdiction of the Supreme Court Constituent assembly. 5. Procedure established by law Select the correct answer using the codes given Q14. Which of the following statements is/are not below: correct? (a) 3 and 4 only 1. The term ‘federation’ has nowhere been used in (b) 2 only the Constitution. (c) 2 and 3 only 2. All India services violate the principle of (d) All of the above federalism under the Constitution. Answer: C Select the correct answer using the codes given Explanation: below: Features adopted from American Constitution (a) 1 only includes following: (b) 2 only . Fundamental rights (c) Both 1 and 2 . Independence of judiciary (d) Neither 1 nor 2 . Judicial review Answer: D . Impeachment of the president. Explanation: The term ‘federation’ has nowhere been used in the Q17. Which among the following was added to the Constitution. Article 1 of the Constitution describes preamble by 42nd Amendment to the Constitution? India as a ‘Union of States’. (a) Liberty of thought and expression . The members of All India services are recruited (b) Socialist, Secular and Integrity and trained by the Centre which also possess (c) Equality of opportunity ultimate control over them. (d) All of the above . Thus, these services violate the principle of Answer: B federalism under the Constitution. Explanation: Preamble has been amended by the 42nd Q15. Modified version of Objective resolution Constitutional Amendment Act (1976), which added forms the three new words—Socialist, Secular and Integrity. (a) Preamble. (b) Fundamental Rights (c) Directive Principles of State policy (d) Fundamental Duties Answer: A Explanation:

Visit: www.civilstap.com For Any Query Mail us: [email protected] or call us at - (+91)-8146207241 33 Subscribe to our YouTube Channel – CivilsTap by EduTap for free, quality and regular content.

Q18. The Government of India Act of 1935 provided (b) 2 and 3 only for (c) 1 only (a) Establishment of an All-India Federation (d) None of the above consisting of provinces and princely States as Answer: A units. Explanation: (b) Separate chapter on Fundamental Rights for The Constitution of India came into force on 26 Indians. January, 1950. (c) Diarchy was introduced in the provinces. . Dr. B.R. Ambedkar is recognised as the ‘Father of (d) It abolished the Council of India, established by the Constitution of India’. the Government of India Act of 1858. Answer: A Q21. Preamble of Indian constitution provides Explanation: insights about following matters Government of India Act of 1935 provided for the (a) Objectives of Indian Constitution as liberty, establishment of an All-India Federation consisting equality and fraternity only. of Provinces and Princely States as units. (b) Nature of Indian State as Sovereign Socialist Secular Democratic Republic. Q19. Consider the following statements: (c) Date of commencement of the Indian 1. Various Indian states had come into an Constitution. agreement to form the Indian federation. (d) All of the above 2. States have right to secede from the federation. Answer: B Which of the statements given above is/are Explanation: Preamble of the Indian Constitution correct? provides insights about: (a) 1 only 1. Source of authority of the Constitution (b) 2 only 2. Nature of Indian State as a Sovereign, Socialist, (c) Both 1 and 2 Secular Democratic and Republican polity. (d) Neither 1 nor 2 3. Objectives of the Indian Constitution as justice, Answer: D liberty, equality and fraternity. Explanation: 4. Date of adoption of the Constitution. In the Indian Constitution the phrase ‘Union of States’ has been preferred to ‘Federation of States’ Q22. Which of the following is/are features of to indicate two things: Indian Constitution? . The Indian federation is not the result of an 1. Federation with a strong Centre. agreement among the states; 2. System of dual citizenship. . The states have no right to secede from the 3. Integrated as well as independent judicial federation. system. 4. Lengthiest constitution of the World at present. Q20. Consider the following statements in the Select the correct answer using the codes given context of Constitution of India: below: 1. The Constitution was adopted on November 26, (a) 3 and 4 only 1949. (b) 2 only 2. Constitution of India came into force on 26 (c) 1, 3 and 4 only November, 1950. (d) All of the above 3. Dr. Rajendra Prasad is recognised as the ‘Father Answer: C of the Constitution of India’ Explanation: Select the correct answer using the codes given Although the Indian Constitution is federal and below: envisages a dual polity it provides for only a single (a) 1 and 2 only citizenship, that is, the Indian citizenship.

Visit: www.civilstap.com For Any Query Mail us: [email protected] or call us at - (+91)-8146207241 34 Subscribe to our YouTube Channel – CivilsTap by EduTap for free, quality and regular content.

Q23. Which of the following statements is/are not Q25. Which of the following statements is/are not correct? correct? 1. From the British side, Cripps Mission (March, 1. Constituent Assembly also worked as the 1942) made the first in principle approval of the provisional Parliament of India with Dr. Rajendra Constituent Assembly for India. Prasad as its elected speaker. 2. Constituent Assembly of India was partly elected 2. Muslim League attended the first meeting of and partly nominated body. Constituent assembly held on December 9, 1946. Select the correct answer using the codes given Select the correct answer using the codes given below: below: (a) 1 only (a) 1 only (b) 2 only (b) 2 only (c) Both 1 and 2 (c) Both 1 and 2 (d) Neither 1 nor 2 (d) Neither 1 nor 2 Answer: A Answer: C Explanation: Explanation: The demand for Constituent Assembly for India was Constituent Assembly also worked as the provisional accepted in principle by the British Government in Parliament of India with G.V. Mavlankar (not Dr. the ‘August Offer’ of 1940. Rajendra Prasad) as its elected speaker. . Members of Provincial Legislative Assembly . Muslim League boycotted the first meeting of elected their representatives in Constituent Constituent Assembly as it was insisted on the Assembly. demand of separate state of Pakistan. . The representatives of Princely states were to be nominated by the heads of the princely states. Q26. Original Constitution of India does not . Thus, the final Constituent Assembly was partly provided for elected and partly nominated body. (a) Preamble (b) Fundamental Rights Q24. Consider the following pairs of adoption date (c) Directive Principles of State policy of various symbol of Indian State and identify the (d) Fundamental Duties ones which are correctly matched: Answer: D 1. National Flag - July 22, 1947 Explanation: 2. National Anthem - January 24, 1950 Original Constitution of India contains the Preamble, 3. National Song - January 26, 1950 fundamental rights & Directive Principles and not Select the correct answer using the codes given the fundamental duties of citizens below: . A set of ten fundamental duties were (a) 1 only incorporated in the Indian Constitution through (b) 2 and 3 only 42nd Constitutional Amendment Act of 1976. (c) 1 and 2 only (d) 3 only Q27. Which of the following is/are the differences Answer: B between the Indian and British Parliamentary Explanation: system of Government? Constituent Assembly of India adopted the national 1. In India head of Government (Prime Minister) flag on July 22, 1947. can be a member of Lower house or Upper . It adopted the national anthem on January 24, house, however, in Britain he should be a 1950. member of Lower house only. . It adopted the national song on January 24, 1950.

Visit: www.civilstap.com For Any Query Mail us: [email protected] or call us at - (+91)-8146207241 35 Subscribe to our YouTube Channel – CivilsTap by EduTap for free, quality and regular content.

2. There is synthesis of Parliamentary Sovereignty (d) Neither 1 nor 2 and Judicial Supremacy in India. Britain follows Answer: C the principle of parliamentary sovereignty only. Explanation: Select the correct answer using the codes given The 73rd Amendment Act of 1992 gave below: constitutional recognition to the panchayats (rural (a) 1 only local governments) by adding a new Part IX and a (b) 2 only new Schedule 11 to the Constitution. Similarly, the (c) Both 1 and 2 74th Amendment Act of 1992 gave constitutional (d) Neither 1 nor 2 recognition to the municipalities. Answer: C Explanation: Q30. Which of the following statements is/are not In Britain, the Prime minister should be a member correct? of the Lower House (House of Commons) of the 1. Sovereignty means the independent authority of Parliament. In India, the Prime minister may be a a state. It has two aspects- external and internal. member of any of the two Houses of Parliament. 2. The word 'democratic' in Indian Preamble refers . The British system is based on the doctrine of to social & economic democracy only. the sovereignty of Parliament. Indian Select the correct answer using the codes given Constitution follows a proper synthesis between below: the principle of parliamentary sovereignty and (a) 1 only the principle of judicial supremacy. (b) 2 only (c) Both 1 and 2 Q28. Which among the following is not a factor (d) Neither 1 nor 2 responsible for the bulkiness of Indian Answer: B Constitution? Explanation: (a) Vast size and large diversity of India The word 'democratic' in Indian Preamble refers to (b) There is single Constitution for Centre as well as political, social and economic democracy. States (c) Due to dominance of lawyers in constituent assembly (d) Legacy of Government of India Act of 1919. Answer: D Explanation: It was the legacy of the Government of India Act of 1935, which became the important reason for the bulkiness of Indian Constitution.

Q29. Consider the following statements: 1. 73rd Constitutional Amendment is about rural local Governments (Panchayati Raj Institutions). 2. 74th Constitutional Amendment made the provisions relating to urban local Government (Municipalities). Which of the statements given above is/are correct? (a) 1 only (b) 2 only (c) Both 1 and 2

Visit: www.civilstap.com For Any Query Mail us: [email protected] or call us at - (+91)-8146207241 36